Exception to second derivative test? (Or am I doing something wrong?)

Click For Summary
The discussion centers on the application of the second derivative test for identifying the nature of stationary points in the function y=(x^2-1)^4. While the test correctly identifies a local maximum at x=0, it fails for x=1 and x=-1, where the second derivative is zero, leading to confusion about their classification. Participants clarify that these points are local minima, not points of inflection, despite the second derivative test suggesting otherwise. The conversation highlights the importance of considering higher-order derivatives when the second derivative is zero. This understanding resolves the initial confusion regarding the nature of the stationary points.
Dramacon
Messages
14
Reaction score
0

Homework Statement


I'd always used the 2nd derivative test for the nature of stationary points. But I came across this equation in one of my exercises, for which the test doesn't seem to work at all.

Find the stationary points of: y=(x^2-1)4, stating the nature of each.

Homework Equations


Using normal calc: the stationary points are at (-1,0), (0,1) and (1,0)

The Attempt at a Solution


Although the double derivative works for when x=0, (local max)

When I sub in the values x=1 or x=-1, the value I end up with is 0: suggesting that these points are stationary points of inflection, when they are not (they're actually local minima).

Please help. :)
 
Physics news on Phys.org
Hi Dramacon! :smile:

From http://en.wikipedia.org/wiki/Inflection_point#A_necessary_but_not_sufficient_condition :wink:

A necessary but not sufficient condition

If x is an inflection point for f then the second derivative, f″(x), is equal to zero if it exists, but this condition does not provide a sufficient definition of a point of inflection. One also needs the lowest-order (above the second) non-zero derivative to be of odd order (third, fifth, etc.). If the lowest-order non-zero derivative is of even order, the point is not a point of inflection. (An example of such a function is y = x4).​
 
Ah, I see! Thank you! :)
 
Question: A clock's minute hand has length 4 and its hour hand has length 3. What is the distance between the tips at the moment when it is increasing most rapidly?(Putnam Exam Question) Answer: Making assumption that both the hands moves at constant angular velocities, the answer is ## \sqrt{7} .## But don't you think this assumption is somewhat doubtful and wrong?

Similar threads

Replies
1
Views
1K
Replies
22
Views
3K
Replies
7
Views
2K
  • · Replies 4 ·
Replies
4
Views
3K
  • · Replies 1 ·
Replies
1
Views
974
  • · Replies 7 ·
Replies
7
Views
2K
  • · Replies 2 ·
Replies
2
Views
2K
  • · Replies 3 ·
Replies
3
Views
2K
Replies
4
Views
2K
  • · Replies 4 ·
Replies
4
Views
2K